What is the solution to the system of equations graphed?​

What Is The Solution To The System Of Equations Graphed?

Answers

Answer 1

Answer:

D. (1, 5)

Step-by-step explanation:

the solution to the system of equations is the point of lines intersect => (1, 5)


Related Questions

Which sign makes the statement true?
5.01 x 10-3 ? 0.00105

Answers

Answer:

47.1 > .00105

Step-by-step explanation:

Given :

5.01×10-3 ? 0.00105

Now,

5.01×10-3 ? 0.00105

50.1-3?0.00105

47.1?0.00105

From the above equation, we can say that 47.1  is greater than 0.00105

Therefore, 47.1 > .00105

Please solve with explanation

Answers

Step-by-step explanation:

total wood= 45½=22.5 units

cut wood= 8⅞= 7 units

wasted wood= 1/16=0.06

now wood left is,

total wood -cut wood-wasted wood

22.5-7-0.06

15.44 units

Hihi , please help if able.

Answers

Answer:

9(9m + 3t) = 81m + 27t

Step-by-step explanation:

With brackets with more than one term inside multiplied by a number;

The rule of thumb to know is that with brackets, everything inside must be multiplied by everything outside;

So, in this case, the number 9 must be multiplied by each term inside the brackets:

9 × 9m = 81m

9 × 3t = 27t

So to expand, opening the brackets, you get:

81m + 27t

Which are the solutions of the quadratic equation?
x² = 7x + 4

Answers

X= 7/2 + 1/2 square root (65)

Answer:

[tex]x = \frac{7 + \sqrt{65}}{2} \ , \ x = \frac{7 - \sqrt{65}}{2}[/tex]

Step-by-step explanation:

[tex]x^2 = 7x + 4 \\\\x^2 - 7x - 4 = 0\\\\ a = 1 \ , \ b = - 7 , \ c \ = \ - 4 \\\\x = \frac{-b \pm \sqt{b^2 - 4ac }}{2a}\\\\Substitute \ the \ values : \\\\x = \frac{7 \pm \sqrt{7^2 - (4 \times 1 \times -4)}}{2 \times 1}\\\\x = \frac{7 \pm \sqrt{49 + 16}}{2 }\\\\x = \frac{7 \pm \sqrt{65}}{2 }\\\\x = \frac{7 + \sqrt{65}}{2}\ , \ x = \frac{7 - \sqrt{65}}{2}[/tex]

Find the value of x in the triangle shown below

Answers

Answer:

x = 48°

Step-by-step explanation:

Two sides of the triangle are equal, both 7, Then the 2 base angles are congruent, both 66°

The sum of the 3 angles in the triangle sum to 180° , so

x = 180° - (66 + 66) = 180° - 132° = 48°

Answer:

[tex]{ \tt{x + 66 + 66 = 180}} \\ x = 48 \degree \\ \\ { \boxed{ \bf{or : }}} \\ { \bf{ from \:lamis \: theorem : }} \\ { \tt{ \frac{5.7}{ \sin(x) } = \frac{7}{ \sin(66 \degree) } }} \\ \\ { \tt{x = { \sin }^{ - 1}( \frac{5.7 \times \sin(66 \degree) }{7} ) }} \\ x = 48 \degree[/tex]

Solve the equation below :

7/8 = q + 1/2

A : q = 3/8


B : q = 1 3/8


C : q = 7/16


D q = 1 3/4

Answers

Answer:

q = -3/8

Step-by-step explanation:

Clearing out the fractions first simplifies this problem.  The LCD here is 8, so we multiply all three terms of 7/8 = q + 1/2 by 8 and simplify the result:

7 + 8q = 4.  

Combining the constants, we get:  8q = 4 - 7, or 8q = -3.

Finally, solve for q:

q = -3/8

I need help on this question, URGENT so please help asap

Answers

Answer: D is correct since the two angles are vertical angles and can be proven congruent by the vertical angle theorem

Step-by-step explanation:

A sphere has a diameter of 32 ft. What is its surface area?
The surface area of the sphere is
ft?. (Type an exact answer in terms of t.)

Answers

Step-by-step explanation:

the answer is in the above image

Answer:  1024pi

======================================================

Explanation:

The diameter 32 cuts in half to 16, which is the radius. So r = 16.

Use this to find the surface area of the sphere in the formula below

SA = 4*pi*r^2

SA = 4*pi*16^2

SA = (4*16^2)*pi

SA = 1024pi

This is the exact surface area in terms of pi.

The area units are in square feet, or ft^2 for short.

Next
Unit Pre Test
Submit Test
20
Type the correct answer in each box. Use numerals instead of words. If necessary, use / for the fraction bar(s)
The equation of a line is 2[v+ 1) = 10x* 3.
The yuntercept of the line is
and the slope of the line is

Answers

Answer:gotta go sorry

Step-by-step explanation:

23. About how much would 4 horses weigh? Write the weight two different ways.​

Answers

An average horse weighs 900-2,000 pounds, depending on size and breed. A lean, racing fit Thoroughbred, for example, has an average weight of 900-1,100 pounds, while the average Clydesdale (think Budweiser) weighs in at 1,800-2,000 pounds

Answer:

If one horse will weigh about 2000 pounds, or 1000kg, then four horses will weigh about 8000 pounds or 4000kg

Find the equation of the line that passes through the point (-5,7) and is perpendicular to the line y=-x+12.

Answers

Answer:

y=x+12

Step-by-step explanation:

y=-x+12; is a line with slope m1=-1

to find perpendicular slope of intersecting line take the negative inverse of m1. so -1*(1/(-1))=1=m2

use equation for a line of y=m*x+b and put in the point (-5,7) and solve for b=the y axis intercept

7=1*(-5)+b

7=-5+b

12=b

so

y=x+12

Select the outlier in the data set.
93
82
10
61
99
89
84
95
75
98
If the outlier were removed from the data set, would the mean increase or decrease?


Answers

Answer:

10

The mean would increase after the outlier is removed

Step-by-step explanation:

An outlier in a dataset is a number that differs significantly from the other data in the set.

In this question, 10 is the number that differs significantly from the other data in the set. Thus, it is the outlier.

Mean = sum of the numbers / total number

Mean including the outlier =  

(93+ 82  + 10  + 61  + 99  + 89  + 84  + 95  + 75  + 98) / 10 = 78.6

Mean without the outlier =

(93+ 82   + 61  + 99  + 89  + 84  + 95  + 75  + 98) / 9 = 86.2

the mean increased after the outlier was removed

determine the equation of the circle graphed below.

( help me please )

Answers

Answer:

(x+5)²+(y-4)²=17

Step-by-step explanation:

I think it's safe to assume that the (-5,4) coordinate is in the center

To find the x and y coordinate just flip the signs

which means it would look like

(x+5)²+(y-4)²=?

the question mark is equal to the raidus squared

to find the radius use the distance formula

√((-4+5)²+(8-4)²)= 4.123106

square this to get 17

the final answer is then

(x+5)²+(y-4)²=17

According to the Rational Root Theorem, -2/5 is a potential rational root of which function? ) = 4x4.72#*#25 O Foxo = 9x47x+10 OF) = 10x - 729 Fox) = 25x4.72​

Answers

Answer:

Option (4)

Step-by-step explanation:

Option (1)

f(x) = 4x⁴- 7x²+ x + 25

Possible rational roots will be,

[tex]\frac{\pm \text{Factors of constant term '25'}}{{\pm \text{Factors of leading coefficient '4'}}}[/tex]

For the given function,

Possible rational roots = [tex]\frac{\pm 1, 5, 25}{\pm 1,2}[/tex]

                                      = [tex]\pm 1, \pm 5, \pm 25, \pm \frac{1}{2},\pm\frac{5}{2},\pm\frac{25}{2}[/tex]

Therefore, [tex]-\frac{2}{5}[/tex] is not the possible root.

Option (2)

f(x) = 9x⁴- 7x²+ x + 10

Possible rational roots = [tex]\frac{\pm 1,\pm 2,\pm 5,\pm10}{\pm 1,\pm3,\pm9}[/tex]

Therefore, [tex]-\frac{2}{5}[/tex] is not the possible root.

Option (3)

f(x) = 10x⁴- 7x²+ x + 9

Possible rational roots = [tex]\frac{\pm1, \pm3, \pm9}{\pm 1,\pm2,\pm5,\pm10}[/tex]

Therefore, [tex]-\frac{2}{5}[/tex] is not the possible root.

Option (4)

f(x) = 25x⁴- 7x²+ x + 4

Possible rational roots = [tex]\frac{\pm 1,\pm2,\pm5}{\pm1,\pm5,\pm 25}[/tex]

                                      = [tex]\pm1,\pm2,\pm5,\pm\frac{1}{5},\pm\frac{1}{25},\pm\frac{2}{5},\pm\frac{2}{25}[/tex]

Therefore, [tex]-\frac{2}{5}[/tex] is the possible rational root.

Option (4) will be the answer.

the sum of three consecutive even intergers is 30. what are the intergers?

Answers

Answer:

8, 10, 12

Step-by-step explanation:

3 consecutive even integers are 8, 10, 12

8 + 10 + 12 = 30.

Answer:

8, 10 ,12

Step-by-step explanation:

Let x be the first integer

x+2 is the second

x+4 is the third

The sum is 30

x+ x+2 + x+4 = 30

Combine like terms

3x+6 = 30

Subtract 6 from each side

3x+6 -6 =30-6

3x = 24

Divide by 6

3x/6 = 24/6

x = 8

x, x+2, x+4  is 8, 8+2, 8+4,

8, 10 ,12

f(x)=15x³+22x²-15x+2
Write f(x) as a product of linear factors.

Answers

Answer:

[tex](5x - 1)(3x - 1)(x + 2)[/tex]

Step-by-step explanation:

[tex](15 {x}^{3} + 22 {x}^{2} - 15x + 2)[/tex]

Apply Rational Root Theorem, our possible roots will be

plus or minus( 2/15, 2/5,2/3,2, 1/15,1/5,1/3,1).

I

I tried root -2 and it work so

If we apply synthetic dividon, we would be left with

[tex]15 {x}^{2} - 8x + 1[/tex]

We can factor this regularly.

Apply AC method that a number

AC will multiply to 15 but add to -8.

The answer are -5 and -3 so we write this as

[tex]15 {x}^{2} - 5x - 3x + 1[/tex]

Factor by grouping

[tex](15x {}^{2} - 5x) - (3x + 1)[/tex]

[tex]5x(3x - 1) - 1(3x - 1)[/tex]

So our factor are

[tex](5x - 1)(3x - 1)(x + 2)[/tex]

What is the common solution for the equations y = 2x + 1 and y = x + 3?

Write your answer an ordered pair!

Answers

Answer:

y^2 = 2x^2+x = x(2x+1) = xy => y^2 = xy => either y=0 or x = y

If y=0 then from y-2x=1, x=-1/2

If X=y then from y-2x = -x = 1 => x=y=-1

Step-by-step explanation:

answer=1

Answer:

(2,5)

Step-by-step explanation:

This is the graph for both equations and the solution is where both lines cross each other.

Hope this helps

Please help me
The question is write the question for the table given.
The multiple choices are
Y=1/3x
Y=1/2x
Y=3x
Y=2x

Answers

Answer:

The answer is Y=3x

when x is 0, y is 0

y=3x

Y= 3 X 0 = 0 (correct)

when x is 1, y is 3

y=3x

y=3 X 1= 3 (correct)

when x is 2 , y is 6

y=3x

y= 3 X 2= 6 (correct)

when x is 3, y is 9

y=3x

y= 3 X 3= 9 (correct)

when x is 4, y is 12

y=3x

y= 3 X 4= 12 ( Correct)

What is the completely factored form of this polynomial?
x^4- 8x^2 + 16
A. (X+2)^2(x-2)^2
B. (x^2-4)^2
C. (x^2+4)(x+2)(x-2)
D. (x+2)(x-2)

Answers

Answer:

A. (x + 2)²(x - 2)²

Step-by-step explanation:

x⁴ - 8x² + 16 =

(x² - 4)(x² - 4) =

(x - 2)(x + 2)(x - 2)(x + 2) =

(x - 2)²(x + 2)²

In the given figure alongside,prove that
Triangle ABC is simalar to Triangle SRT
Find the length of AC​

Answers

Answer:

let's use Pythagoras theorem,

h²=b²+l²

so,

ad²=ac²+dc²

6²=ac²+3²

36-9=ac²=27

√27 can be written as 3√3,

hence ac= 3√3

Help please no links

Answers

Answer:

D

Step-by-step explanation:

The question states you need a square. That means whatever you do, must leave a square number like 49 or 64

If you add 6 or 7 to 56  you do not get 64    A and C are both wrong.

If you subtract 7 from 56 you get 49 which is a perfect square and all 4 sides will equal 7.

Sora paid $26.46 for 8.4 gallons of gasoline. How much was each gallon of gasoline?
$0.211
$0.315
$2.11
$3.15

Answers

Answer:

last one

Step-by-step explanation:

The answer is $3.15 I got this by dividing 26.46 by 8.4

one dozen mangoes cost $120.00. what is the cost of 8 mangoes??​

Answers

Answer:

$80.00

Step-by-step explanation:

12m = 120

m = 10

Therefore, 8m must equal 80.

Answer:

The cost of 8 mangos would be $80.00

Step-by-step explanation:

A dozen is equal to 12 so its 10 bucks per mango since there are 12 mangos

Which of the following is NOT true about mathematical induction?

A.The first possible case is always n = 1.

B.Mathematical induction depends on a recursive process.

C.It can be used to prove that 1 + 2 + 3+...+n =
n(n+2)
2

D. Since Sn is valid for n = 1, it is valid for n = 2. Since it is valid for n = 2, it is valid for n = 3, and so on, indefinitely.​

Answers

Answer:

A. the first possible case is always n = 1

Step-by-step explanation:

Mathematical induction is a technique used to provide proof for a statement such that the statement holds for all natural numbers which are the non-negative integers

Therefore, given that the natural numbers are 0, 1, 2..., we have that mathematical induction can start from n = 0

Therefore, the statement which is not true is that the first possible case is always n = 1

Which is greater, 2 miles or 1,000 yards? How much greater? Explain. Of 2 miles and 1,000 yards, _____ is greater. Since 2 miles is the same as _____ yards, _____ is __ yards greater than _____ . ​

Answers

Answer:

yards

Step-by-step explanation:

Answer:

Which is greater, 2 miles

1 mile = 1760 yards

2 miles = 2 * 1760

2 miles = 3,520 yards

How much greater?

3,520 - 1000 = 2520 yards

In the circle below, segment AB is a diameter. If the length of are ACB is 6pi what is the length of the radius of the circle?

Answers

Answer:

The radius is 6

Step-by-step explanation:

Arc length ACB = 6 pi

The arc length = fraction of a circle times the circumference

6 pi = 180/360 * 2 * pi *r

6 pi = 1/2 * 2 * pi*r

6 pi =  pi r

The radius is 6

I need the answers and it's due today, please help

Answers

Answer:

1. 3

2. 1

3. 2

4. 4

5. 5

Step-by-step explanation:

Answer plsssss………………

Answers

Answer:

The corresponding angles theorem works for cases where we have two parallel lines intersecting another line.

Two lines are parallel if, at any point, the distance between these two lines is always the same.

Now, if we look at the image, we can see that the distance between the two horizontal lines changes (is smaller at the right and larger at the left)

Thus, these lines are not parallel.

Then the corresponding angles theorem can not be used here, and we have that:

∠9 ≠ ∠10

Surface area is the sum of the areas of all the surfaces of a three-dimensional object.
(True or false)

Answers

Answer: True

Think of a 3D box. It has 6 faces and the surface area is the total of all six individual rectangular faces. This idea applies to any polyhedron.

The quartile deviation and coefficient of quartile deviation of a continuous frequency distribution are 2 and 0.25 respectively. Find lower and upper quartiles.

Answers

Answer:

Lower quartile = 6

Uppwr quartile = 10

Step-by-step explanation:

Coefficient of quartile deviation = 0.25

Quartile deviation = 2

Coefficient of quartile deviation = (Q3 - Q1) / (Q3 + Q1)

Quartile deviation = (Q3 - Q1) / 2

Hence;

Quartile deviation = (Q3 - Q1) / 2 = 2

Q3 - Q1 = 2 * 2

Q3 - Q1 = 4 - - - - (1)

Q3 = 4 + Q1 - - - - (2)

(Q3 - Q1) / (Q3 + Q1) = 0.25

Q3 - Q1 = 4

4 = 0.25(Q3 + Q1)

Q3 + Q1 = 4 / 0.25

Q3 + Q1 = 16 - - - - (3)

Put Q3 = 4 + Q1 in (3)

4 + Q1 + Q1 = 16

4 + 2Q1 = 16

2Q1 = 16 - 4

Q1 = 12 / 2

Q1 = 6

Q3 = 4 + Q1

Q3 = 4 + 6

Q3 = 10

Other Questions
in a market economy decisions about which goods are produced are based on What happens to the particle motion of a substance if the temperature changes? Which expression are greater than 1/2? Choose all the apply How much is six dimes, 8 nickels, and three one-dollar bills?* The base of an open rectangular box is of length (2x + 5) cm and width x cm.The area of this base is 58 cmThe height of the open box is (x - 2) cm.a) Show that 2x + 5x -58 = 0) Solve the equation 2x + 5x -58 = 0, giving your answers correctto 2 decimal placesD) Hence calculate the volume of the box, stating the units of your answer. The endosymbiosis hypothesis is supported by all of the following pieces of evidence except the fact that Group of answer choices chloroplasts reproduce through a splitting process like certain prokaryotes. mitochondria have circular DNA like prokaryotes. chloroplasts have ribosomes like prokaryotes. mitochondria lack ribosomes like prokaryotes. idioms of unforgettable due today pls helppppp Define and explain SMART? what is 69/2 as a mixed number HELP ME PLEASE!!!Which 2 statements are true about this chemical reaction that forms acid rain? which sultan of ottoman empire was beautiful ?kosem sultan Hrrem Sultan A motor boat travels 60 miles down a river in 3 hours but takes 5 hours to return upstream. Find the rate of the boat in still water and the rate of the current. Hi savaine this is for you A card is picked from a standard deck of 52 cards. Determine the odds against and the odds in favor of selecting a 4. NEED HELP; A group of friends goes out to dinner. The total bill, without tax, is $63. If tax on the meal is 6% and if an 18% tip is left on the $63 total bill, what is the total amount paid?A. $78.12B. $78.80C. $66.78D. $74.34 The Scottsboro Boys were: Take a look at the two boxes below. Each box has the same volume. If each ball has the same mass, which box 1 or 2 would weigh more? Why? Type the correct answer in the box.Consider the table below. -3 0.5-21-1 2.50518.5Complete the standard form equation representing the quadratic relationship displayed above, where a, b, and care constants. Which angle would be complementary to